Đề thi và đáp án kì thi chọn đội tuyển thi Quốc gia trường Phổ thông Năng khiếu năm học 2019 – 2020

ĐỀ THI

Ngày thi thứ nhất

Bài 1. Số thực $\alpha$ được gọi là điểm tụ của dãy số $\left(u_n\right)$ nếu tồn tại ít nhất một dãy con của $\left(u_n\right)$ có hội tụ đến $\alpha$.

(a) Hãy chỉ ra một dãy số có vô hạn điểm tụ.

(b) Chứng minh rằng nếu dãy số có mọi dãy con hội tụ thì nó cũng hội tụ.

(c) Gọi $S$ là tập hợp tất cả các số chính phương dương. Dãy số $\left(a_n\right)$ xác định bởi $a_n=\frac{1}{n}$ nếu $n \in S$ và $a_n=\frac{1}{n^2}$ nếu $n \notin S$.

Đặt $b_n=\sum_{k=1}^n a_k$. Xét tính hội tụ của các dãy số $\left(a_n\right)$ và $\left(b_n\right)$.

Bài 2. Tìm tất cả các hợp số dương $n$ sao cho $\sigma(n) \equiv 2(\bmod \varphi(n))$, trong đó ký hiệu $\sigma(n), \varphi(n)$ là hàm tổng các ước của $n$ và hàm Euler.

Bài 3. Tìm tất cả các hàm số $f: \mathbb{R} \rightarrow \mathbb{R}$ thỏa mãn

$\quad\quad\quad\quad\quad f(f(x)+y)+f(x) f(f(y))=x f(y)+x+y, \forall x, y \in \mathbb{R} .$

Bài 4. Cho tam giác $A B C$ không cân nội tiếp trong đường tròn $(O)$ với $B C$ cố định và $A$ thay đổi trên cung lớn $B C$. Các đường tròn bàng tiếp góc $A, B, C$ lần lượt tiếp xúc với $B C, C A, A B$ tại $D, E, F$. Gọi $L, M, N$ lần lượt là giao điểm khác $A, B, C$ của $(A B E),(A C F) ;(B C F),(B A D) ;(C A D),(C B E)$.

(a) Chứng minh rằng $A L$ luôn đi qua điểm cố định khi $A$ thay đổi.

(b) Gọi $K, I, J$ lần lượt là trung điểm của $A D, B E, C F$. Chứng minh rằng $K L, I M, J N$ dồng quy.

Ngày thi thứ hai

Bài 5. Cho $a, b, c$ là các số thực dương thỏa mãn $8\left(a^2+b^2+c^2\right)=9(a b+b c+c a)$.

Tìm giá trị lớn nhất và giá trị nhỏ nhất của biểu thức

$\quad\quad\quad\quad\quad\quad\quad\quad\quad\quad T=\frac{a+b}{c}+\frac{b+c}{a}+\frac{c+a}{b} .$

Bài 6. Tìm tất cả các hàm số $f: \mathbb{Z}^{+} \rightarrow \mathbb{Z}^{+}$thỏa mãn đồng thời các điều kiện sau

$\quad\quad$ i) $m f(m)+n f(n)+2 m f(n)$ là số chính phương với mọi $m, n$;

$\quad\quad$ ii) $f(m n)=f(m) f(n)$ với mọi $m, n$ nguyên dương;

$\quad\quad$  iii) Với mọi số nguyên tố $p, f(p)$ không chia hết cho $p^2$.

Bài 7. Một trường phổ thông có $n$ học sinh. Các học sinh tham gia vào tổng cộng $m$ câu lạc bộ là $A_1, A_2, \ldots, A_m$.

(a) Chứng minh rằng nếu mỗi câu lạc bộ có 4 học sinh và hai học sinh bất kỳ tham gia chung nhất một câu lạc bộ thì $m \leq \frac{n(n-1)}{12}$.

(b) Giả sử tồn tại $k>0$ sao cho hai câu lạc bộ bất kỳ có chung nhau $k$ thành viên và tồn tại một câu lạc bộ $A_t$ có $k$ thành viên. Chứng minh rằng $m \leq n$

Bài 8. Cho tam giác $A B C$ nội tiếp đường tròn $(O)$. Đường tròn nội tiếp $(I)$ tiếp xúc với các cạnh $B C, C A, A B$ lần lượt tại $D, E, F$. Gọi $J$ là tâm bàng tiếp góc $A$ của tam giác $A B C$ và $H$ là hình chiếu của $D$ lên $E F$.

(a) Chứng minh rằng giao điểm của $A H, J D$ thì thuộc đường thẳng $O I$.

(b) Giả sử $D H$ cắt lại $(I)$ ở $K$ và $I K$ cắt lại đường tròn ngoại tiếp $(I E F)$ ở $L$. Chứng minh rằng $A D, L H$ cắt nhau tại một điểm nằm trên $(I E F)$.

 

LỜI GIẢI

Ngày thi thứ nhất

Bài 1. Số thực $\alpha$ được gọi là điểm tụ của dãy số $\left(u_n\right)$ nếu tồn tại ít nhất một dãy con của $\left(u_n\right)$ có hội tụ đến $\alpha$.

(a) Hãy chỉ ra một dãy số có vô hạn điểm tụ.

(b) Chứng minh rằng nếu một dãy số có mọi dãy con hội tụ thì nó cũng hội tụ.

(c) Gọi $S$ là tập hợp tất cả các số chính phương dương. Dãy số $\left(a_n\right)$ xác định bởi $a_n=\frac{1}{n}$ nếu $n \in S$ và $a_n=\frac{1}{n^2}$ nếu $n \notin S$.

Đặt $b_n=\sum_{k=1}^n a_k$. Xét tính hội tụ của các dãy số $\left(a_n\right)$ và $\left(b_n\right)$.

Lời giải. (a) Ta sẽ chỉ ra dãy số mà mỗi số nguyên dương xuất hiện vô hạn lần trong đó. Chẳng hạn $\left(u_n\right)$ : là

$\quad\quad\quad\quad\quad\quad\quad\quad\quad\quad 1,2,1,2,3,1,2,3,4,1,2,3,4,5, \ldots$

với $u_n=1$ nếu $n \in S$ và $u_{n+1}=u_n+1$ nếu $n \notin S$, trong đó $S$ là tập hợp các số có dạng $\frac{m(m+1)}{2}$ như $1,3,6,10,15, \ldots$ Khi đó, với mỗi số nguyên dương $m \in \mathbb{Z}^{+}$thì ta luôn có thể trích ra một dãy con vô hạn của $\left(u_n\right)$ có tất cả các phần tử đều bằng $m$, tức là hội tụ về $m$.

(b) Do mỗi dãy số là dãy con của chính nó nên rõ ràng khẳng định của bài toán là đúng.

(c) Ta có $0 \leq a_n \leq \frac{1}{n}$ với mọi $n$ nên theo nguyên lí kẹp, ta suy ra $\lim a_n=0$. Nhận xét rằng $b_n$ là dãy tăng. Ta có

$\quad\quad\quad\quad\quad b_{n^2}=\sum_{i=1}^{n^2} a_i =\sum_{i \in S, i=1}^{n^2} a_i+\sum_{i \notin S, i=1}^{n^2} a_i=\sum_{i \in S, i=1}^{n^2} \frac{1}{i}+\sum_{i \notin S, i=1}^{n^2} \frac{1}{i^2} $

$\quad\quad\quad\quad\quad\quad\quad\quad\quad\quad\quad =\left(1+\frac{1}{2^2}+\cdots+\frac{1}{n^2}\right)+\sum_{i \notin S, i=1}^{n^2} \frac{1}{i^2} $

$\quad\quad\quad\quad\quad\quad\quad\quad\quad\quad\quad <\left(1+\frac{1}{2^2}+\cdots+\frac{1}{n^2}\right)+\left(\sum_{i=1}^{n^2} \frac{1}{i^2}\right) $

$\quad\quad\quad\quad\quad\quad\quad\quad\quad\quad\quad =\left(1+\frac{1}{2^2}+\cdots+\frac{1}{n^2}\right)+\left(1+\frac{1}{2^2}+\cdots+\frac{1}{n^4}\right)$

Vì dãy $u_n=1+\frac{1}{2^2}+\cdots+\frac{1}{n^2}<1+\frac{1}{1 \cdot 2}+\frac{1}{2 \cdot 3}+\cdots+\frac{1}{(n-1) n}=2-\frac{1}{n}<2$ là bị chặn trên nên từ đánh giá đã xây dựng được, ta có $b_{n^2}$ cũng bị chặn trên. Kết hợp với $b_n$ là dãy tăng, ta suy ra bản thân dãy $b_n$ cũng bị chặn trên nên nó hội tụ.

Bài 2. Tìm tất cả các hợp số dương $n$ sao cho

$\quad\quad\quad\quad\quad\quad\quad\quad\quad\quad \sigma(n) \equiv 2 \quad(\bmod \varphi(n)),$

trong đó ký hiệu $\sigma(n), \varphi(n)$ là hàm tổng các ước của $n$ và hàm Euler.

Lời giải . Giả sử $p$ là một ước nguyên tố lẻ của $n$. Nếu $v_p(n)>1$ thì theo công thức của hàm Euler, ta có $p \mid \varphi(n)$, mà $n \cdot \sigma(n)-2$ chia hết cho $\varphi(n)$, tức là cũng chia hết cho $p$ nên kéo theo $p \mid 2$, vô lý. Suy ra $v_p(n)=1$ với mọi $p \mid n$.

Đặt $n=2^k \cdot p_1 p_2 \ldots p_t$ với $k \geq 0$ và $p_1<p_2<\ldots<p_t$ là các số nguyên tố phân biệt. Theo công thức tính các hàm, ta có

$\quad\quad\quad\quad\quad\quad\quad \varphi(n)=2^{k-1}\left(p_1-1\right)\left(p_2-1\right) \ldots\left(p_t-1\right)$

$\quad\quad\quad\quad\quad\quad\sigma(n)=\left(2^{k+1}-1\right)\left(p_1+1\right)\left(p_2+1\right) \ldots\left(p_t+1\right) .$

Đánh giá lũy thừa 2 trong các số trên, ta có

$\quad\quad\quad\quad\quad\quad v_2(\varphi(n)) \geq k-1+t \text { và } v_2(n \cdot \sigma(n)) \geq k+t .$

Do đó từ $\varphi(n) \mid n \cdot \sigma(n)-2$, ta suy ra $1 \geq k-1+t$ nên $k+t \leq 2$. Ta xét các trường hợp sau

  • Nếu $t=0$ thì $n=2^k$ là hợp số nên $k=2, n=4$, thử trực tiếp ta thấy thỏa.
  • Nếu $t=1$ thì $n=2 p$ nên $\varphi(n)=p-1, \sigma(n)=3(p+1)$ và đưa về

$\quad\quad\quad\quad\quad\quad\quad\quad\quad\quad p-1 \mid 6 p(p+1)-2$

Chú ý rằng

$\quad\quad\quad\quad\quad\quad 6 p(p+1)-2=6 p^2+6 p-2=(p-1)(6 p+12)-10$

nên $p-1 \mid 10$. Từ đó ta tìm được $p=3, p=11$ tương ứng với $n=6, n=22$.

  • Nếu $t=2$ thì $k=0$, ta có $n=p_1 p_2$ nên

$\quad\quad\quad\quad \varphi(n)=\left(p_1-1\right)\left(p_2-1\right) \text { và } \sigma(n)=\left(p_1+1\right)\left(p_2+1\right)$

đưa về

$\quad\quad\quad\quad \left(p_1-1\right)\left(p_2-1\right) \mid\left(p_1+1\right)\left(p_2+1\right)-2 .$

Điều này không thể xảy ra vì $\left(p_1-1\right)\left(p_2-1\right)$ chia hết cho 4 trong khi biểu thức còn lại chia 4 dư 2 . Do đó, trường hợp này không có số $n$ thỏa mãn.

Vậy tất cả các số cần tìm là $4,6,22$.

Nhận xét. Chú ý rằng mọi số nguyên tố đều thỏa mãn yêu cầu của đề bài.

Bài 3. Tìm tất cả các hàm số $f: \mathbb{R} \rightarrow \mathbb{R}$ thỏa mãn

$\quad\quad\quad\quad\quad\quad f(f(x)+y)+f(x) f(f(y))=x f(y)+x+y$

với mọi số thực $x, y$.

Lời giải. Thay $x=y=0$ vào phương trình đề cho, ta có

$\quad\quad\quad\quad\quad\quad\quad\quad\quad\quad f(f(0))+f(0) f(f(0))=0 .$

suy ra $f(f(0))=0$ hoặc $f(0)=-1$. Ta xét các trường hợp sau:

  1. Nếu $f(f(0))=0$. Thay $y=0$, vào phương trình dề cho, ta có $f(f(x))=$ $x f(0)+x, \forall x \in \mathbb{R}$ Thay $x=f(0)$ và sử dụng $f(f(0))=0$, ta được $f(0)=$ $[f(0)]^2+f(0)$, hay $f(0)=0$. Do đó $f(f(x))=x$ với mọi $x \in \mathbb{R}$. Thay vào phương trình đề bài, ta có

$\quad\quad\quad\quad\quad\quad f(f(x)+y)+y f(x)=x f(y)+x+y, \forall x, y \in \mathbb{R} .$

Thay $y$ bởi $f(y)$ và sử dụng tính đối xứng của vế trái, ta được

$\quad\quad\quad\quad\quad\quad f(f(x)+f(y))+f(x) f(y)=x y+x+f(y)=x y+y+f(x) .$

Do đó $f(x)-x=f(y)-y$ với mọi $x, y \in \mathbb{R}$, hay $f(x)=x+c$. Thử lại, ta có $c=0$.

  1. Nếu $f(0)=-1$. Thay $y=0$ vào phương trình đề cho, ta có $f(f(x))+$ $f(x) f(-1)=0, \forall x \in \mathbb{R}$. Từ đây suy ra $f(f(-1))=-[f(-1)]^2$. Thay $x=0$ vào phương trình đề cho, ta có $f(y-1)-f(f(y))=y, \forall y \in \mathbb{R}$. Kết hợp các đẳng thức trên lại, ta có

$\quad\quad\quad\quad\quad\quad\quad\quad f(x-1)+f(x) f(-1)=x, \forall x \in \mathbb{R} .$

Thay $y=-1$ vào phương trình đề cho và sử dụng $f(f(-1))=-[f(-1)]^2$, ta lại có

$\quad\quad\quad\quad\quad\quad f(f(x)-1)-f(x)[f(-1)]^2=x f(-1)+x-1, \forall x \in \mathbb{R} .$

Mặt khác, ta cũng có

$\quad\quad\quad\quad\quad\quad\quad\quad f(-1) f(f(x))+f(x)[f(-1)]^2=0, \forall x \in \mathbb{R} .$

Cộng vế theo vế hai biểu thức trên lại, ta có

$\quad\quad\quad\quad\quad\quad\quad\quad\quad\quad f(x)=[1+f(-1)] x+1, \forall x \in \mathbb{R} .$

Thử lại, ta thấy không thỏa mãn.

Vậy phương trình có nghiệm hàm duy nhất là $f(x)=x$.

Bài 4. Cho tam giác $A B C$ không cân nội tiếp trong đường tròn $(O)$ với $B C$ cố định và $A$ thay đổi trên cung lớn $B C$. Các đường tròn bàng tiếp góc $A, B, C$ lần lượt tiếp xúc với các cạnh $B C, C A, A B$ tại $D, E, F$. Gọi $L, M, N$ lần lượt là giao điểm khác $A, B, C$ của các cặp đường tròn

$\quad\quad\quad\quad (A B E),(A C F) ;(B C F),(B A D) ;(C A D),(C B E) .$

(a) Chứng minh rằng $A L$ luôn đi qua điểm cố định khi $A$ thay đổi.

(b) Gọi $K, I, J$ lần lượt là trung điểm của $A D, B E, C F$. Chứng minh rằng $K L, I M, J N$ đồng quy.

Lời giải . (a) Đặt $B C=a, C A=b, A B=c$ và $p$ là nửa chu vi thì theo tính chất tiếp điểm bàng tiếp, ta có $B F=C E=p-a$.

Bằng biến đổi góc, ta có được $\triangle L B F \sim \triangle L E C(g . g)$, mà $B F=C E$ nên hai tam giác này bằng nhau. Suy ra $L B=L E, L C=L F$ nên $L$ là trung điểm cung $B E$ của đường tròn $(A B E)$ và cũng là trung diểm cung $C F$ của $(A C F)$.

Từ đó ta có $A L$ là phân giác góc $B A C$ hay $A L$ luôn đi qua trung điểm cung nhỏ $B C$ của $(O)$, là điểm cố định.

(b) Để ý rằng vai trò của $M, N, L$ là bình đẳng trong tam giác $A B C$. Do đó, từ câu a, một cách tương tự, ta có $M, N$ thuộc phân giác góc $B, C$ nên cũng lần lượt là trung điểm các cung nhỏ của các đường tròn tương ứng. Suy ra $M, K, N$ thẳng hàng (cùng thuộc trung trực của đoạn $A D$ ); tương tự với các bộ ba $N, I, L$ và $L, J, M$. Cuối cùng, ta thấy rằng

$\quad\quad\quad\quad\quad\quad\quad \frac{K M}{K N}=\frac{A K \cdot \tan \angle M A K}{A K \cdot \tan \angle N A K}=\frac{\tan (B / 2)}{\tan (C / 2)} .$

Tương tự với các tỷ số khác. Đến đây, áp dụng định lý Ceva cho tam giác $M N L$, ta có các đoạn thẳng $L K, I M, J N$ dồng quy.

Nhận xét. Một cách khác cho câu a như sau: Xét phép nghịch đảo đối xứng với phương tích $k=A B \cdot A C$ và trục đối xứng là phân giác góc $A$. Ta có $E \rightarrow E^{\prime} \in$ $A C, F \rightarrow F^{\prime} \in A B$ sao cho $A E \cdot A E^{\prime}=A F \cdot A F^{\prime}=k$. Ta tính được

$\quad\quad\quad\quad A E^{\prime}=\frac{b c}{p-c} \rightarrow B E^{\prime}=\frac{c(p-a)}{p-c} \rightarrow \frac{E^{\prime} B}{E^{\prime} A}=\frac{p-a}{b} .$

Tương tự thì $\frac{F^{\prime} C}{F^{\prime} A}=\frac{p-a}{c}$. Áp dụng định lý Ceva cho tam giác $A B C$ thì $C E^{\prime}, B F^{\prime}$ và phân giác góc $A$ đồng quy.

Lại có qua phép nghịch đối xứng trên thì phân giác giữ nguyên,

$\quad\quad\quad\quad\quad\quad\quad\quad\quad\quad (A B E) \rightarrow C F^{\prime},(A C F) \rightarrow B E^{\prime}$

nên ta có $L$ thuộc phân giác góc $A$.

 

Ngày thi thứ hai

Bài 5. Cho $a, b, c$ là các số thực dương thỏa mãn $8\left(a^2+b^2+c^2\right)=9(a b+b c+c a)$. Tìm giá trị lớn nhất và giá trị nhỏ nhất của biểu thức

$\quad\quad\quad\quad\quad\quad\quad\quad\quad\quad T=\frac{a+b}{c}+\frac{b+c}{a}+\frac{c+a}{b} .$

Lời giải . Do tính thuần nhất đối xứng của các biến nên chuẩn hóa

$\quad\quad\quad\quad\quad a b+b c+c a=8 \rightarrow a^2+b^2+c^2=9 \rightarrow a+b+c=5 .$

Ta có $P+3=(a+b+c)\left(\frac{1}{a}+\frac{1}{b}+\frac{1}{c}\right)=\frac{40}{a b c}$ nên ta đưa về tìm min, max của $T=a b c$ trong điều kiện

$\quad\quad\quad\quad\quad\quad\quad\quad\quad\quad\quad \left\{\begin{array}{l}a+b+c=5 \\ a b+b c+c a=8\end{array}\right.$

Chú ý rằng $b+c=5-a, b c=8-a(b+c)=8-a(5-a)$ nên từ đánh giá quen thuộc $(b+c)^2 \geq 4 b c$, ta có

$\quad\quad\quad\quad\quad\quad (5-a)^2 \geq 4\left(8-5 a+a^2\right) \Leftrightarrow 1 \leq a \leq \frac{7}{3} .$

Suy ra $T=a b c=a\left(8+a^2-5 a\right)=f(a)$. Đến đây khảo sát hàm số này trên miền $\left[1 ; \frac{7}{3}\right]$, ta được $\min T=4, \max T=\frac{112}{27}$ nên $\min P=\frac{93}{14}$, $\max P=7$. Từ đó, ta thu được kết luận như sau

  • Giá trị lớn nhất của $P$ là 7 , đạt được chẳng hạn khi $(a, b, c)=(2,2,1)$.
  • Giá trị nhỏ nhất của $P$ là $\frac{93}{14}$, đạt được chẳng hạn khi $(a, b, c)=\left(\frac{7}{3}, \frac{4}{3}, \frac{4}{3}\right)$.

Bài 6. Tìm tất cả các hàm số $f: \mathbb{Z}^{+} \rightarrow \mathbb{Z}^{+}$thỏa mãn đồng thời các điều kiện sau đây

$\quad\quad$ i) $m f(m)+n f(n)+2 m f(n)$ là số chính phương với mọi $m, n$;

$\quad\quad$  ii) $f(m n)=f(m) f(n)$ với mọi $m, n$ nguyên dương;

$\quad\quad$  iii) Với mọi số nguyên tố $p, f(p)$ không chia hết cho $p^2$.

Lời giải . Thay $m=n=1$ vào ii), ta suy ra $f(1)=f(1)^2$ nên $f(1)=1$. Thay $m=n$ vào i), ta suy ra $4 m f(m)$ là số chính phương với mọi $m \in \mathbb{Z}^{+}$nên $m f(m)$ cũng là số chính phương với mọi $m \in \mathbb{Z}^{+}$.

Với $p$ là số nguyên tố, vì $p f(p)$ là số chính phương nên $p \mid f(p)$ và ta đặt $f(p)=k^2 p$, với $k$ là số nguyên dương nào đó. Thay $m=p, n=1$ vào i), ta suy ra $p f(p)+1+2 p$ là số chính phương, hay $k^2 p^2+2 p+1$ là số chính phương.

Vì $k^2 p^2+2 p+1>(k p)^2$ nên

$\quad\quad\quad\quad\quad\quad k^2 p^2+2 p+1 \geq(k p+1)^2=k^2 p^2+2 k p+1 .$

Do đó $2 p \geq 2 k p$ nên ta phải có $k=1$.

Vì thế nên $f(p)=p$ với mọi số nguyên tố $p$. Sử dụng điều kiện ii), hàm $f$ nhân tính, và cũng vì mọi số nguyên dương bất kỳ đều có thể viết dưới dạng tích của các số nguyên tố nên ta có được $f(n)=n$ với mọi $n \in \mathbb{Z}^{+}$.

Thử lại ta thấy hàm số này thỏa mãn các ràng buộc của đề bài.

Nhận xét. Trên thực tế, ta có thể bỏ bớt diều kiện ii), iii) đi mà bài toán gốc vẫn có thể giải quyết được. Cụ thể như sau:

Chứng minh rằng nếu hàm số $f: \mathbb{Z}^{+} \rightarrow \mathbb{Z}^{+}$thỏa mãn $f(1)=1$ và với mọi $m, n \in \mathbb{Z}^{+}$, ta có $m f(m)+n f(n)+2 m f(n)$ là số chính phương thì $f(n)=n, \forall n \in \mathbb{Z}^{+}$.

Bài 7. Một trường phổ thông có $n$ học sinh. Các học sinh tham gia vào tổng cộng $m$ câu lạc bộ là $A_1, A_2, \ldots, A_m$.

(a) Chứng minh rằng nếu mỗi câu lạc bộ có 4 học sinh và hai học sinh bất kỳ tham gia chung nhất một câu lạc bộ thì $m \leq \frac{n(n-1)}{12}$.

(b) Giả sử tồn tại $k>0$ sao cho hai câu lạc bộ bất kỳ có chung nhau $k$ thành viên và tồn tại một câu lạc bộ $A_t$ có $k$ thành viên. Chứng minh rằng $m \leq n$

Lời giải . (a) Gọi $S$ là số bộ $({A, B}, C)$ mà trong đó học sinh $A, B$ cùng tham gia vào câu lạc bộ $C$. Ta thực đếm $S$ bằng hai cách

  1. Chọn câu lạc bộ trước, có $m$ cách, chọn cặp học sinh cùng tham gia vào đó có $C_4^2=6$ cách nên $S=6 \mathrm{~m}$.
  2. Chọn cặp học sinh trước, có $C_n^2$ cách, chọn câu lạc bộ mà hai học sinh đó cùng tham gia, có không quá 1 cách nên $S \leq C_n^2$.

Từ đó suy ra

$\quad\quad\quad\quad\quad\quad\quad\quad\quad 6 m \leq C_n^2 \Leftrightarrow m \leq \frac{n(n-1)}{12} .$

(b) Xét câu lạc bộ $X$ nào đó có $k$ thành viên. Xét $m-1$ câu lạc bộ còn lại thì theo giả thiết, rõ ràng các câu lạc bộ này đều có chứa $k$ thành viên trên của câu lạc bộ $X$. Từ đó suy ra $m-1$ câu lạc bộ còn lại đôi một không có thành viên chung.

Xét $n-k$ học sinh còn lại của trường thì rõ ràng một học sinh thuộc tối đa một câu lạc bộ (trong số các câu lạc bộ còn lại), suy ra số câu lạc bộ còn lại không vượt quá $n-k$ nên suy ra $m \leq n-k+1 \leq n$. Ta có điều phải chứng minh.

Nhận xét. Ý b của bài toán khá hiển nhiên, nhưng thực ra nó là một “phiên bản dễ” của bất đẳng thức Fisher sau đây:

Cho $A_1, A_2, \ldots, A_m$ là các tập con của tập ${1,2, \ldots, n}$ sao cho hai tập con bất kỳ có chung nhau đúng $k$ (với $k$ là số nguyên cố định nào đó không vượt quá n). Khi đó $m \leq n$.

Tuy nhiên, chứng minh sơ cấp cho kết quả này quả thực rất khó. Cách phổ biến nhất là dùng đại số tuyến tính. Cụ thể là:

Ta đặt tương ứng mỗi tập $A_i$ với một vector $v_i$ trong $\mathbb{F}_2^n$ như sau

$\quad\quad\quad\quad\quad\quad\quad\quad v_{i j}=\left\{\begin{array}{l}1 \text { nếu } j \in A_i \\ 0 \text { nếu } j \notin A_i\end{array} .\right.$

Chú ý rằng $\left|A_i \cap A_j\right|=k$ với mọi $i \neq j$. Bởi vậy, các vector $v_1, \ldots, v_m$ là các vector trong $\mathbb{R}^n$. Mặt khác, ta có số chiều của $\mathbb{R}^n$ là $n$. Do đó, trong bước tiếp theo chúng ta chỉ cần chứng minh $v_1, \ldots, v_m$ độc lập tuyến tính trong không gian $\mathbb{R}^n$.

Giả sử phản chứng rằng tồn tại các hệ số $\alpha_1, \ldots \alpha_m$ không đồng nhất bằng không sao cho $\sum_{i=1}^m \alpha_i v_i=0$. Do đó, ta có

$\quad\quad\quad\quad\quad\quad 0 =\left|\sum_{i=1}^m \alpha_i v_i\right|^2=\left\langle\sum_{i=1}^m \alpha_i v_i, \sum_{i=1}^m \alpha_i v_i\right\rangle $

$\quad\quad\quad\quad\quad\quad\quad =\sum_{i=1}^m \alpha_i^2\left|v_i\right|^2+\sum_{1 \leq i \neq j \leq m} \alpha_i \alpha_j\left\langle v_i, v_j\right\rangle$

Mặt khác, $\left|v_i\right|^2=\left|A_i\right|$, và $\left\langle v_i, v_j\right\rangle=\left|A_i \cap A_j\right|$. Bởi vậy,

$\quad\quad\quad\quad 0=\sum_{i=1}^m \alpha_i^2\left|v_i\right|^2+\sum_{i \neq j} k \alpha_i \alpha_j=\sum_{i=1}^m \alpha_i^2\left(\left|A_i\right|-k\right)+k \sum_{1 \leq i, j \leq m} \alpha_i \alpha_j$

Ta thấy rằng $\sum_{1 \leq i, j \leq m} \alpha_i \alpha_j=\left(\sum_{1 \leq i \leq m} \alpha_i\right)^2$, nên $0=\sum_{i=1}^m \alpha_i^2\left(\left|A_i\right|-k\right)+k\left(\sum_{1 \leq i, j \leq m} \alpha_i\right)^2$. Vì $\left|A_i\right| \geq k$ và có nhiều nhất một tập với chính xác $k$ phần tử, nên $\alpha_1=\cdots=\alpha_m=0$.

Điều này mâu thuẫn với giả thiết, hay các vector $v_1, \ldots, v_m$ là độc lập tuyến tính. Như vậy ta sẽ có $m \leq n$.

Bài 8. Cho tam giác $A B C$ nội tiếp đường tròn $(O)$. Đường tròn nội tiếp $(I)$ tiếp xúc với các cạnh $B C, C A, A B$ lần lượt tại $D, E, F$. Gọi $J$ là tâm bàng tiếp góc $A$ của tam giác $A B C$ và $H$ là hình chiếu của $D$ lên $E F$.

(a) Chứng minh rằng giao điểm của $A H, J D$ thì thuộc đường thẳng $O I$.

(b) Giả sử $D H$ cắt lại $(I)$ ở $K$ và $I K$ cắt lại đường tròn ngoại tiếp $(I E F)$ ơ $L$. Chứng minh rằng $A D, L H$ cắt nhau tại một diểm nằm trên $(I E F)$.

Lời giải. (a) Ta có bổ đề sau:

Bổ ĐỀ. $O I$ là đường thẳng Euler của tam giác $D E F$.

Bổ đề này quen thuộc và có thể chứng minh bằng cách hướng như sau (chi tiết xin dành cho bạn đọc).

  1. Sử dụng phép nghịch đảo tâm $I$, phương tích $r^2$ biến $(O)$ thành đường tròn Euler của $D E F$ nên có các tâm thẳng hàng.
  2. Sử dụng phép vị tự bằng cách gọi thêm trung điểm các cung nhỏ $B C, C A, A B$ của $(O)$.

Khi đó, gọi $T$ là giao điểm của $I O$ và $H D$ thì rõ ràng $T$ là trực tâm của tam giác $D E F$. Gọi $M$ là trung điểm cung nhỏ $B C$ của $(O)$ thì dễ thấy $M$ là trung điểm $I J$.

Bằng biến đổi góc, ta có $\triangle T E F \sim \triangle I B C$, mà $T H, I D$ là hai đường cao tương ứng nên $\frac{T H}{I D}=\frac{E F}{B C}$. Mặt khác, $\triangle I E F \sim \triangle M B C$ nên

$\quad\quad\quad\quad\quad\quad\quad\quad \frac{E F}{B C}=\frac{I E}{M C}=\frac{2 I E}{2 M I}=\frac{2 I E}{I J}$

suy ra $\frac{T H}{I D}=\frac{2 I E}{I J}$. Do đó

$\quad\quad\quad\quad\quad\quad T H \cdot I J=2 I D^2=2 I N \cdot I A=T D \cdot I A$

(vì $I, T$ lần lượt là tâm ngoại tiếp và trực tâm tam giác $D E F$ ) nên $\frac{T H}{T D}=\frac{I A}{I J}$. Cuối cùng, vì $H D | A J$ (cùng vuông góc với $E F$ ) nên theo định lý Talet thì $A H, J D, T I$ đồng quy hay nói cách khác, $A H, J D$ cắt nhau trên $O I$.

(b) Giả sử $A D$ cắt lại $(I)$ tại $G$. Ta cần chứng minh rằng $G, H, L$ thẳng hàng.

Do $D K | A I$ nên

$\quad\quad\quad\quad\quad\quad\quad\quad\quad\quad\angle A G L=\angle A I L=\angle A I K=\angle D K I$

suy ra $\angle D G L=\angle D K L$. Vì thế nên $D G K L$ là tứ giác nội tiếp. Do đó, $L G$ là trục đẳng phương của $(L K D),(I E F)$. Lại có

nên suy ra $H$ thuộc trục đẳng phương của hai đường tròn này, tức là $H \in L G$. Từ đó ta có điều phải chứng minh.

Nhận xét. Liên quan đến ý b, có một bài toán khá thú vị với nội dung như sau:

Trung tuyến đỉnh $D$ của tam giác $D E F$ cắt $(I)$ ở $L$. Chứng minh rằng trục đẳng phương của $(L B F),(L C E)$ đi qua giao điểm của $J D$ và đường thẳng qua $A$, vuông góc với $A I$.

 

 

 

 

 

 

 

 

 

 

 

 

 

 

 

 

 

 

 

 

 

 

 

 

 

 

 

 

 

 

 

 

 

 

 

 

 

 

 

 

 

 

 

 

 

 

 

 

 

 

 

 

 

 

 

 

 

 

 

 

 

 

 

 

 

 

 

 

 

 

 

 

 

 

 

 

 

 

 

 

 

 

 

 

 

 

 

 

 

 

 

 

 

 

 

 

 

 

 

 

Leave a Reply

Your email address will not be published. Required fields are marked *